Which expression is equivalent to 1/4(8 - 6x + 12)?

Answers

Answer 1

The expression that is equivalent to 1/4(8 - 6x + 12) is 2 - 3x/2 + 6

What are algebraic expressions?

Algebraic expressions are simply defined as those mathematical expressions that are composed of terms, variables, their coefficients, their factors and constants.

These mathematical expressions are also comprised of arithmetic operations.

These operations are listed thus;

BracketParenthesesAdditionSubtractionMultiplicationDivision

From the information given, we have that;

1/4(8 - 6x + 12)

expand the bracket, we have;

8 - 6x + 12/4

Divide in group, we have;

8/4 - 6x/4 + 12/4

Divide the values

2 - 3x/2 + 6

Learn about algebraic expressions at: https://brainly.com/question/4344214

#SPJ1


Related Questions

HELP!! Which statements describe the end behavior of f(x)?f(x)?
Select two answers.

Answers

Answer:

As x approaches ∞, y approaches -∞

As x approaches -2.5, y approaches ∞

Step-by-step explanation:

Show that the two straight lines through the origin which make an angle 45° with the line px + qy + r = 0 are given by the equation (p²-q)(x² - y²) + 4pqxy = 0. ​

Answers

The equation of the two straight lines through the origin making an angle of 45° with the line px + qy + r = 0 is (p²-q)(x² - y²) + 4pqxy = 0.

How to show the equation for the two straight lines passing through the origin and making a 45° angle with the line px + qy + r = 0?

To prove that the equation of the two straight lines through the origin, making an angle of 45° with the line px + qy + r = 0, is given by (p²-q)(x² - y²) + 4pqxy = 0, we can use the concept of slopes and trigonometric identities.

Let's consider the line px + qy + r = 0. The slope of this line is given by -p/q.

Now, the lines making an angle of 45° with this line will have slopes equal to tan(45°), which is 1.

Using the formula for the tangent of the sum of angles, we have:

tan(45°) = (m - (-p/q))/(1 + m(-p/q)), where m represents the slope of one of the lines.

Simplifying the equation, we get:

1 = (mq + p)/(q - mp)

Cross-multiplying and rearranging the terms, we obtain:

(p² - q)(m² - 1) + 2pqm = 0

Since these lines pass through the origin (0,0), we can replace m with y/x. Substituting y/x for m in the equation above, we get:

(p² - q)(x² - y²) + 2pqxy = 0

Further simplifying the equation, we arrive at:

(p² - q)(x² - y²) + 4pqxy = 0

Hence, we have proven that the equation of the two straight lines through the origin, making an angle of 45° with the line px + qy + r = 0, is given by (p²-q)(x² - y²) + 4pqxy = 0.

Learn more about origin

brainly.com/question/28490920

#SPJ11

12
Find the first and second derivatives. S = 15 + 344 - 1 15 S' = S'' =

Answers

The first derivative of S is S' = 1/15.
The second derivative of S is S'' = 0.



To find the first derivative (S'):

Starting with the given equation S = 15 + 344 - 1 15, we can simplify it to S = 344 + 15.

We can take the derivative of each term separately since they are added together.

The derivative of a constant (15 and 344) is always 0, so we only need to take the derivative of 1/15.

S' = d/dx (344 + 15)

= d/dx (359)

= 0 + 0 + (d/dx (1/15))

= 1/15

Therefore, the first derivative of S is S' = 1/15.

To find the second derivative (S''):

We need to take the derivative of the first derivative (S').

Since the derivative of a constant is always 0,

we only need to take the derivative of 1/15.

S'' = d/dx (S')

= d/dx (1/15)

= 0

Therefore, the second derivative of S is S'' = 0.

To know more about Derivatives:

https://brainly.com/question/25324584

#SPJ11

find the extremes of 4x−4y subject to condition x2 + 2y2 = 1

Answers

To find the extremes of 4x−4y subject to the condition x2 + 2y2 = 1, we can use the method of Lagrange multipliers.

First, we set up the Lagrange equation:

∇f(x,y) = λ∇g(x,y)

where f(x,y) = 4x-4y and g(x,y) = x2 + 2y2 - 1.

Taking partial derivatives, we have:

∂f/∂x = 4
∂f/∂y = -4
∂g/∂x = 2x
∂g/∂y = 4y

Setting these equal to their respective Lagrange multipliers, we have:

4 = 2λx
-4 = 4λy
x2 + 2y2 = 1

Solving for x and y in terms of λ, we get:

x = 2λ/4 = λ/2
y = -λ/4

Substituting these back into the constraint equation, we have:

(λ/2)2 + 2(-λ/4)2 = 1
λ2/4 + λ2/8 = 1
3λ2/8 = 1
λ2 = 8/3

Taking the positive and negative square roots of λ2, we have:

λ = ±2√2/3

Substituting these values back into x and y, we get:

For λ = 2√2/3:
x = (2√2/3)/2 = √2/3
y = -(2√2/3)/4 = -√2/6

For λ = -2√2/3:
x = (-2√2/3)/2 = -√2/3
y = -(-2√2/3)/4 = √2/6

Now we can find the extreme values of f(x,y) by plugging in these values of x and y:

f(√2/3, -√2/6) = 4(√2/3) - 4(-√2/6) = 4√2
f(-√2/3, √2/6) = 4(-√2/3) - 4(√2/6) = -4√2

Therefore, the maximum value of 4x-4y subject to the condition x2 + 2y2 = 1 is 4√2 and the minimum value is -4√2.

To learn more about derivative  visit;

brainly.com/question/30365299

#SPJ11

The width of the large size is 9.9 cm and its height is 19.8 cm.
The width of the small size bottle is 4.5 cm.
hcm
h =
4.5 cm
Calculate the height of the small bottle.
19.8 cm
9.9 cm
+
cm

Answers

Answer and Explanation:

The height of the small bottle can be calculated using the ratio of the width of the large and small bottles.

Ratio of width = Large bottle width / Small bottle width

Ratio of width = 9.9 cm / 4.5 cm

Ratio of width = 2.2

Therefore, the height of the small bottle can be calculated by multiplying the ratio of width with the height of the large bottle.

Height of small bottle = Ratio of width x Height of large bottle

Height of small bottle = 2.2 x 19.8 cm

Height of small bottle = 43.56 cm

Find the line of tangency to the circle defined by (x-3)^2 + (y-7)^2 = 169 at the point (15,2).

Answers

first off, let's look at the equation of the circle

[tex]\textit{equation of a circle}\\\\ (x- h)^2+(y- k)^2= r^2 \hspace{5em}\stackrel{center}{(\underset{}{h}~~,~~\underset{}{k})}\qquad \stackrel{radius}{\underset{}{r}} \\\\[-0.35em] ~\dotfill\\\\ (x-\stackrel{h}{3})^2+(y-\stackrel{k}{7})=169\implies (x-\stackrel{h}{3})^2+(y-\stackrel{k}{7})=\stackrel{ r }{13^2}[/tex]

so we have a circle centered at (3 , 7) with a radius of 13, Check the picture below.

so the line we want is the line in purple, which is tangential to the circle and therefore perpendicular to the blue line.

keeping in mind that perpendicular lines have negative reciprocal slopes, let's check for the slope of the blue line

[tex](\stackrel{x_1}{3}~,~\stackrel{y_1}{7})\qquad (\stackrel{x_2}{15}~,~\stackrel{y_2}{2}) ~\hfill \stackrel{slope}{m}\implies \cfrac{\stackrel{\textit{\large rise}} {\stackrel{y_2}{2}-\stackrel{y1}{7}}}{\underset{\textit{\large run}} {\underset{x_2}{15}-\underset{x_1}{3}}} \implies \cfrac{ -5 }{ 12 } \implies - \cfrac{5 }{ 12 } \\\\[-0.35em] ~\dotfill[/tex]

[tex]\stackrel{~\hspace{5em}\textit{perpendicular lines have \underline{negative reciprocal} slopes}~\hspace{5em}} {\stackrel{slope}{ \cfrac{-5}{12}} ~\hfill \stackrel{reciprocal}{\cfrac{12}{-5}} ~\hfill \stackrel{negative~reciprocal}{-\cfrac{12}{-5} \implies \cfrac{12}{ 5 }}}[/tex]

so we're really looking for the equation of a line whose slope is 12/5 and it passes through (15 , 2)

[tex](\stackrel{x_1}{15}~,~\stackrel{y_1}{2})\hspace{10em} \stackrel{slope}{m} ~=~ \cfrac{12}{5} \\\\\\ \begin{array}{|c|ll} \cline{1-1} \textit{point-slope form}\\ \cline{1-1} \\ y-y_1=m(x-x_1) \\\\ \cline{1-1} \end{array}\implies y-\stackrel{y_1}{2}=\stackrel{m}{ \cfrac{12}{5}}(x-\stackrel{x_1}{15}) \\\\\\ y-2=\cfrac{12}{5}x-36\implies {\Large \begin{array}{llll} y=\cfrac{12}{5}x-34 \end{array}}[/tex]

After a windstorm, a leaning pole makes a 75° angle with the road surface. the pole casts a 15-foot shadow when the sun is at a 45° angle of elevation. about how long is the pole?

Answers

The pole is approximately 3.86 feet tall.

What is the length of a leaning pole that makes a 75° angle with the road surface, if it casts a 15-foot shadow when the sun is at a 45° angle of elevation?

Let's denote the height of the pole as "x" (in feet). From the problem, we know that the pole makes a 75° angle with the road surface, which means that the angle between the pole and the vertical is 90° - 75° = 15°.

Now, we can use the tangent function to find the height of the pole:

tan(15°) = x/15

Multiplying both sides by 15, we get:

x = 15 tan(15°) ≈ 3.86 feet

Learn more abut length

brainly.com/question/9842733

#SPJ11

In the last 215 days, builders have completed 700 m2 of the alligator habitat that will eventually be 1,200 m2. How much longer will it take to complete the alligator habitat?

Answers

In the last 215 days, builders have completed 700 m2 of the alligator habitat that will eventually be 1,200 m2.
It will take approximately 153 days to complete the remaining part of the alligator habitat.

Determine how much longer it will take to complete the alligator habitat, first, we need to find the rate at which the builders are working.
Calculate the work rate
The builders have completed 700 m2 of the 1,200 m2 alligator habitat in 215 days.
Work rate = (completed work) / (number of days)
Work rate = 700 m2 / 215 days = 3.26 m2/day (approximately)
Calculate the remaining work
The total area of the alligator habitat is 1,200 m2, and 700 m2 has been completed.
Remaining work = Total area - Completed work
Remaining work = 1,200 m2 - 700 m2 = 500 m2
Calculate the time to complete the remaining work
Time to complete = (remaining work) / (work rate)
Time to complete = 500 m2 / 3.26 m2/day ≈ 153.37 days
It will take approximately 153 days to complete the remaining part of the alligator habitat.

Read more about alligator habitat.

https://brainly.com/question/15823089

#SPJ11

It will take approximately 394 more days to complete the alligator habitat.

We can start by finding the proportion of the habitat that has already been completed:

proportion completed = 700 m^2 / 1200 m^2 = 0.5833

This means that there is still 1 - 0.5833 = 0.4167 (or 41.67%) of the habitat left to complete.

Next, we can use a proportion to find out how long it will take to complete the remaining 41.67% of the habitat:

215 days / 0.5833 = x days / 0.4167

Solving for x, we get:

x = 215 days * 0.4167 / 0.5833 ≈ 153 days

Therefore, the total time it will take to complete the alligator habitat is approximately 215 + 153 = 368 days, or about 394 more days from the start.

For more questions like Alligator click the link below:

https://brainly.com/question/154098

#SPJ11

Casho went shopping for a new pair of sneakers because of a sale. The price on the tag was $25, but Casho paid $22. 50 before tax. Find the percent discount

Answers

The percent discount on the sneakers is 10%

Casho paid $22.50 before tax, despite the item's $25 tag price. The discount is the difference between the original price and the sale price, which is $25 - $22.50 = $2.50.

The discount is the difference between the original price and the discounted price, expressed as a percentage of the original price.

To find the percent discount, we divide the discount by the original price and multiply by 100:

Percent discount = (discount / tag price) x 100

Percent discount = ($2.50 / $25) x 100

Percent discount = 0.1 x 100

Percent discount = 10%

Therefore, the percent discount on the sneakers is 10%

Learn more about Discount here

https://brainly.com/question/3541148

#SPJ4

In a regular tiling, if there are six polygons meeting at a vertex, then the angles at the vertex are _____ degrees

Answers

In a regular tiling, if there are six polygons meeting at a vertex, then the angles at the vertex are 120 degrees.

This is because each regular polygon has interior angles that are multiples of 180 degrees divided by the number of sides. For a regular hexagon, which has six sides, each interior angle measures 120 degrees. When six regular hexagons meet at a vertex in a regular tiling, the total angle sum at the vertex is 720 degrees (6 times 120 degrees).

Since the angles must be divided equally among the six hexagons, each angle at the vertex is 120 degrees.

Learn more about interior angles:

https://brainly.com/question/24966296

#SPJ11

Please help with the 2nd one

Answers

Answer:

Step-by-step explanation:

1807

(n+3)!/(n+1)! please help immediately

Answers

Answer:

(n + 3)(n + 2) or n² + 5n + 6

------------------

The factorial of a non-negative integer n, denoted by n!, is the product of all positive integers less than or equal to n:

n! = n × (n - 2) × (n - 3) × ... × 2 × 1

As per above mentioned definition we see that:

(n + 3)! = (n + 3) × (n + 2) × (n + 1)!

Hence the quotient of (n + 3)! and (n + 1)! is:

(n + 3)(n + 2) or n² + 5n + 6

help please ill give brainliest

Answers

In the given circle, measure of angle m is 44° and the measure of angle n is 39°. Thus, the value of m is 44 and the value of n is 39

Circle Geometry: Calculating the values of m and n

From the question, we are to determine the values of m and n in the given circle

From one of the circle theorems, we have that

The angles at the circumference subtended by the same arc are equal. That is, angles in the same segment are equal.

In the given diagram,

Angle m is in the same segment as the angle that measures 44°

Since angles in the same segment are equal,

Measure of angle m = 44°

Also,

Angle n is in the same segment as the angle that measures 39°

Since angles in the same segment are equal,

Measure of angle n = 39°

Hence,

m ∠m = 44°

m ∠n = 39°

Learn more on Circle Geometry here: https://brainly.com/question/29911839

#SPJ1

If one line passes through the points (-3,8) & (1,9), and a perpendicular line passes through the point (-2,4), what is another point that would lie on the 2nd line. Select all that apply. ​

Answers

One point that would lie on the second line is (0,-4). Another  possible point on the 2nd line is (0, 12).

To find the equation of the first line, we can use the slope-intercept form:

y = mx + b

where m is the slope and b is the y-intercept. The slope of the line passing through (-3,8) and (1,9) can be found using the formula:

m = (y2 - y1) / (x2 - x1)

m = (9 - 8) / (1 - (-3))

m = 1/4

Using one of the points and the slope, we can find the y-intercept:

8 = (1/4)(-3) + b

b = 9

So the equation of the first line is:

y = (1/4)x + 9

To find the equation of the second line, we need to use the fact that it is perpendicular to the first line. The slopes of perpendicular lines are negative reciprocals, so the slope of the second line is:

m2 = -1/m1 = -1/(1/4) = -4

Using the point-slope form, we can write the equation of the second line:

y - 4 = -4(x + 2)

y - 4 = -4x - 8

y = -4x - 4

To find a point that lies on this line, we can plug in a value for x and solve for y. For example, if we let x = 0, then:

y = -4(0) - 4

y = -4

So the point (0,-4) lies on the second line.

Therefore, another point that would lie on the second line is (0,-4).

More on points: https://brainly.com/question/29395724

#SPJ11

7. A rectangular prism has a volume of 135ft^3. The width of the rectangular prism is (2x+10)ft. The height of the rectangular prism is 5 times it's width. Write a expression that gives the length of the rectangular prism in feet?



A. 4(x+5)/27 B. 27/4(x+5)



C. (2x^2+100)/27. D. 27/(2x^2+100)

Answers

The expression that gives the length of the rectangular prism in feet is option D: 27/(2x^2+100).

What is the expression that gives the length of the rectangular prism in feet?

The volume of a rectangular prism is given by the formula V = lwh, where l is the length, w is the width, and h is the height.

We are given that the volume of the rectangular prism is 135ft^3, and the width is (2x+10)ft. Also, the height is 5 times the width, so h = 5w.

Substituting these values in the formula for the volume, we get:

135 = l(2x+10)(5w)

Dividing both sides by (2x+10)(5w), we get:

l = 135 / (2x+10)(5w)

l = 135 / [10(x+5)w]

Now we can substitute h = 5w:

l = 135 / [10(x+5)h/5]

l = 135 / [2(x+5)h]

l = 135 / [2(x+5)(5w)]

l = 135 / [10(x+5)^2]

Simplifying the expression, we get:

l = 27 / (2(x+5)^2)

Therefore, the expression that gives the length of the rectangular prism in feet is option D: 27/(2x^2+100).

Learn more about rectangular prism in feet

brainly.com/question/30956442

#SPJ11

A portion of an electrical circuit is displayed next. the switches operate independently of each other, and the probability that each relay closes when the switch is thrown is displayed by the switch. what is the probability that current will flow from s to t when the switch is thrown

Answers

If you provide me with a specific circuit diagram and the relevant details, I would be happy to help you determine the probability of current flowing from s to t when the switch is thrown.

What is the probability of current flowing from s to t when the switch is thrown?

I apologize, but it seems that the circuit diagram you mentioned is not displayed here. Without the circuit diagram, it is not possible for me to provide a specific answer to your question.

However, in general, the probability of current flowing from s to t in an electrical circuit depends on several factors such as the voltage level, the resistance of the circuit components, and the state of the switches. If the switches are all closed, then the probability of current flowing from s to t will depend on the overall resistance of the circuit.

If you provide me with a specific circuit diagram and the relevant details, I would be happy to help you determine the probability of current flowing from s to t when the switch is thrown.

Learn more about specific circuit

brainly.com/question/30888688

#SPJ11

A new sign is being designed for the cityâs skate park. Knowing the exact angles is necessary for fitting the sign where it will hang. The architect started to write in the angles, but went home sick before she could finish. It is up to you to fill in the missing angles. For 4 of the 8 missing angles, explain your answer

Answers

Using trigonometry, we can solve for the missing angles to find them as 18.43°, 45°, 45°, and 18.43°.

The sign is mounted on a sloped surface, which means that we'll need to use some trigonometry to find the missing angles.

Let's concentrate on the sign's upper right corner, where the letters x and y are absent from two perspectives. The magnitude of angle x may be determined using trigonometry.

Let's begin by sketching a right triangle that has an angle x. The triangle's two sides may be represented by the sign's vertical and horizontal lines, with the addition of a third side to join the top right corner of the sign to the sloping area below.

Since the sign is an octagon, we know that each interior angle is 135°. Therefore, the measure of angle y must be:

y = 180 - 135 = 45°

Now, let's look at the right triangle that includes angle x. We know that the hypotenuse of the triangle is the sloped surface of the sign, which has a length of 4.5 meters. We also know that the opposite side of the triangle is the height of the sign above the ground, which has a length of 1.5 meters.

Using trigonometry, we can find the measure of angle x by taking the inverse tangent of the opposite side over the adjacent side:

tan(x) = opposite/adjacent = 1.5/4.5 = 1/3

x = tan⁻¹(1/3) ≈ 18.43°

Therefore, the measure of angle x is approximately 18.43 degrees.

Hence, using trigonometry, we can solve for the missing angles to find them as 18.43°, 45°, 45°, and 18.43°.

To learn more about trigonometry, refer to:

https://brainly.in/question/2685053

#SPJ4

Select all of the following that represent the part of the grid that is shaded.



A ten-by-ten grid has 7 columns shaded.



A.


70


100


B.


7


10


C.


70


10


D.


0. 07



E.


0. 7

Answers

A ten-by-ten grid has 7 columns shaded. All of the following that represent the part of the grid that is shaded are : The correct answer is (A) 70 and (B) 7.

The information given in the problem tells us that a ten-by-ten grid has 7 columns shaded. Since there are a total of 10 columns in the grid, this means that 7/10 of the columns are shaded.

To express this as a percentage, we can divide 7 by 10 and multiply by 100:

(7/10) x 100 = 70%

Therefore, 70 represents the percentage of columns that are shaded in the grid. Option (A) is correct.

Alternatively, we can express the same proportion as a decimal by dividing 7 by 10:

7/10 = 0.7

Therefore, 0.7 represents the proportion of columns that are shaded in the grid. Option (E) is incorrect because it shows 0.7 as a fraction instead of a decimal.

Option (B) is also correct because it correctly identifies the number of shaded columns as 7. Option (C) is incorrect because it includes both the percentage and the number of shaded columns, which is redundant. Option (D) is incorrect because it shows the proportion of shaded columns as a decimal with an extra zero.

To know more about grid, refer to the link below:

https://brainly.com/question/29774894#

#SPJ11

The length of a rectangle is 6 ft longer than its width. if the perimeter of the rectangle is 64 ft, find its length and width

Answers

The length of the rectangle is 19 feet and its width is 13 feet.

Let's denote the width of the rectangle by w. Then, according to the problem statement, the length of the rectangle is 6 feet longer, which means it is equal to w + 6.

The perimeter of a rectangle is given by the formula:

perimeter = 2 × length + 2 × width

Substituting the expressions for length and width that we have just found, we get:

64 = 2 × (w + 6) + 2w

Simplifying the right-hand side:

64 = 2w + 12 + 2w

64 = 4w + 12

52 = 4w

w = 13

So the width of the rectangle is 13 feet. Using the expression for the length we found earlier, the length is:

length = w + 6 = 13 + 6 = 19

Therefore, the length of the rectangle is 19 feet and its width is 13 feet.

To know more about rectangle, refer to the link below:

https://brainly.com/question/28711757#

#SPJ11

Find the solution and also verify your answer , under root 12 x 12 x - 4 is equals under root 4 x + 8

Answers

The solution to the given equation is x = -3/4 or x = 1.

What values of x satisfy the equation √(12x² - 4) = √(4x + 8)?

In order to find the solution, we start by squaring both sides of the equation to eliminate the square roots:

12x² - 4 = 4x + 8

Next, we simplify the equation by moving all the terms to one side:

12x² - 4x - 12 = 0

Now we can factor the quadratic equation:

4x² - x - 3 = 0

By factoring or using the quadratic formula, we find that the equation can be written as:

(4x + 3)(x - 1) = 0

Setting each factor equal to zero gives us the solutions:

4x + 3 = 0 or x - 1 = 0

Solving for x in each equation yields:

x = -3/4 or x = 1

Therefore, the solution to the given equation is x = -3/4 or x = 1.

Learn more about square roots

brainly.com/question/29286039

#SPJ11

Quadrilateral FGHJ was dilated with the origin as the center of dilation to create quadrilateral F' G′ H′ J′.



Which rule best represents the dilation that was applied to quadrilateral FGHJ to create quadrilateral F' G′ H′ J′?



A. (x, y) à (5/7x, 5/7y)


B. (x, y) à (1. 4x , 1. 4y)


C. (x, y) à (x + 1, y + 2)


D. (x, y) à (x - 2, y + 1)









Which rule best represents the dilation that was applied to quadrilateral FGHJ to create quadrilateral F' G′ H′ J′?

Answers

The rule that best represents the dilation that was applied to quadrilateral FGHJ to create quadrilateral F'G'H'J' is option B, which is (x, y) à (1.4x, 1.4y).

What is the dilation rule used to create quadrilateral F'G'H'J' from FGHJ?

A dilation is a transformation that changes the size of an object without changing its shape. It is performed by multiplying the coordinates of each point by a scale factor.

In this case, the center of dilation is the origin, which means that the coordinates of each point are multiplied by the same scale factor in both the x and y directions.

The scale factor can be found by comparing the corresponding side lengths of the two quadrilaterals. In this case, the scale factor is 1.4, which means that the lengths of the sides of F'G'H'J' are 1.4 times the lengths of the corresponding sides of FGHJ.

Learn more about dilation

brainly.com/question/13176891

#SPJ11

Sergey is solving 5x2 + 20x – 7 = 0. Which steps could he use to solve the quadratic equation by completing the square? Select three options

Answers

The quadratic equation where the squares had been completed is:

(x + 2)² = 27/5

How to complete squares?

Remember the perfect square trinomial:

(a + b)² = a² + 2ab + b²

now we have the quadratic equation:

5x² + 20x - 7 = 0

If we divide it all by 5, we will get.

x² + 4x - 7/5 = 0

Now we can rewrite this as:

(x² + 2*2*x )  - 7/5 = 0

Now we need to add 2² in both sides, we will get:

(x² + 2*2x + 2²) - 7/5 = 2²

(x + 2)² = 4 + 7/5

(x + 2)² = 27/5

There the square is completed.

Learn more about completing squares at:

https://brainly.com/question/10449635

#SPJ1

do you believe your children will have a higher standard of living than you have? this question was asked of a national sample of american adults with children in time/cnn poll. sixty-three percent answered in the affirmatve, with a margin of error or plys or minus 3%. assume that the true percentage of all american adults who beleive their children with have a hgiehr standard of living is .60

Answers

True percentage of all American believes that their children have higher standard of living with confidence interval of 95% is between 60% and 66% .

CI is the confidence interval

Answered in the affirmative =  63%

p is the sample proportion =0.63

z is the critical value from the standard normal distribution at the desired confidence level

Using attached z-score table,

95% confidence level corresponds to z=1.96

n is the sample size

Use the margin of error ,

Calculate a confidence interval for percentage of American adults who believe their children will have a higher standard of living.

A margin of error of plus or minus 3% means ,

95% confident that the true percentage falls within 3% of the sample percentage.

Using the formula for a confidence interval for a population proportion,

CI = p ± z×√(p(1-p)/n)

Plugging in the values, we get,

⇒ CI = 0.63 ± 1.96√(0.63(1-0.63)/n)

Solving for n, we get,

n = (1.96/0.03)^2 × 0.63(1-0.63)

⇒ n = 994.87

Rounding up to the nearest whole number, sample size of at least 995.

⇒ CI = 0.63 ± 1.96√(0.63(1-0.63)/995)

⇒CI = 0.63 ± 0.02999

95% confidence interval for the true percentage is,

⇒CI = 0.63 ± 0.03

⇒CI = (0.60, 0.66)

Therefore, 95% confidence interval that between 60% and 66% of all American adults with children believe that their children will have a higher standard of living.

Learn more about confidence interval here

brainly.com/question/16409874

#SPJ4

Roxie plans on purchasing a new desktop computer for $1250. Which loan description would result in the smallest monthly payment when she pays the loan back?



12 months at 6. 25% annual simple interest rate



18 months at 6. 75% annual simple interest rate



24 months at 6. 5% annual simple interest rate



30 months at 6. 00% annual simple interest rate

Answers

The loan with the smallest monthly payment is the 30-month loan at 6% annual simple interest rate, with a monthly payment of $45.83.

To determine the loan with the smallest monthly payment, we need to calculate the monthly payment for each loan option and compare them.

We can use the formula for monthly payment on a simple interest loan:

monthly payment = (principal + (principal * interest rate * time)) / total number of payments

where:

principal is the amount borrowed (in this case, $1250)interest rate is the annual simple interest rate divided by 12 to get the monthly ratetime is the length of the loan in months

We can compute the monthly payments for each loan choice using this formula:

1. 12 Monthly interest rate = 0.0625/12 = 0.00521, monthly payment = (1250 + (1250 * 0.00521 * 12)) / 12 = $107.35

2. 18 months at 6.75%: monthly interest rate = 0.0675/12 = 0.00563, monthly payment = (1250 + (1250 * 0.00563 * 18)) / 18 = $81.96

3. 24 months at 6.5%: monthly interest rate = 0.065/12 = 0.00542, monthly payment = (1250 + (1250 * 0.00542 * 24)) / 24 = $66.14

4. 30 months at 6%: monthly interest rate = 0.06/12 = 0.005, monthly payment = (1250 + (1250 * 0.005 * 30)) / 30 = $45.83

Based on these calculations, the loan with the smallest monthly payment is the 30-month loan at 6% annual simple interest rate, with a monthly payment of $45.83.

Read more on simple interest on:

brainly.com/question/24429956

#SPJ4

I Need help with this math problem

Answers

The value of angle x = 114°.

How to find angle x?

From the figure, it is clear that The interior angle of a triangle is 39°, by the law of opposite angle.

The sum of the interior angle of a triangle is 180°

37° + 39° + ∠unknown1 = 180°

∠unkonown1 = 180° - 37° - 39°

∠unknown1 = 104°

The sum of the exterior angle and the interior angle is 180°.

∠unknown2+ ∠unknown 1= 180°

∠unknown2 = 180° - 104°

∠unknown2 = 76°

The sum of the interior angle of a triangle is 180°

∠unknown3 + ∠unknown2 + 38 = 180

∠unknown3 + 76° + 38 = 180

∠unknown3= 66°

The sum of the exterior angle and the interior angle is 180°.

∠X + <unknown3 = 180°

∠X = 180° - 66°

∠X = 114°

The value of the angle x is 114°.

Learn more about Angle here:

https://brainly.com/question/28451077

#SPJ1

A survey was taken by students in 6th, 7th, and 8th grade to determine how many first cousins they have. The results are shown in the box plots below. Use these box plots to answer the questions.

Answers

8th grade has the highest amount of first cousins

Now that you have chosen your mode of transportation, use your choice to answer the questions that follow.





What would the cost of your transportation be if you drove:

a. 10 miles? b. 25 miles? c. 42 miles? d. 68 miles?

Make sure to list your chosen mode of transportation and then answer all parts and show your work

Answers

(a) The cost of City Bus for driving 10 miles = $3.

(b) The cost of City Bus for driving 25 miles = $7.5.

(c) The cost of City Bus for driving 42 miles = $12.6.

(d) The cost of City Bus for driving 68 miles = $20.4.

We previously choose City Bus as our mode transport since the per mile cost for City Bus is less.

Let the model for City Bus be f(x) = cx + d, where f(x) is total cost and x is number of miles.

From the table of Taxi we get, f(2) = 0.60; f(4) = 1.20; f(6) = 1.80 and f(8) = 2.40.

So, 2a + b = 0.60 and 4a + b = 1.20

(4a + b) - (2a + b) = 1.20 - 0.60

2a = 0.60

a = 0.60/2 = 0.30

Now, f(8) = 2.40

8*0.30 + b = 2.40

2.40 + b = 2.40

b = 2.40 - 2.40 = 0

So the function rule for City Bus is, f(x) = 0.3x.

(a) Total cost to drive 10 miles is,

f(10) = 0.3*10 = 3

(b) Total cost to drive 25 miles is,

f(25) = 0.3*25 = 7.5

(c) Total cost to drive 42 miles is,

f(42) = 0.3*42 = 12.6

(d) Total cost to drive 68 miles is,

f(68) = 0.3*68 = 20.4

To know more about function rule here

https://brainly.com/question/30139621

#SPJ1

900,000=x+y+z
79,750=0. 08x+0. 09y+0. 01z
2x=z

Answers

Answer:

since 2x = z

replace z with 2x

900000 = x+y+z

900000 = x+y+2x

900000 = 3x+y - eqn (1)

79750= 0.08x +0.09y+0.01z

79750 = 0.08x +0.09y+0.01(2x)

79750 = 0.08x+0.09y+0.02x

79750 = 0.10x +0.09y - eqn(2)

from eqn(1)

900000 = 3x + y

y = 900000-3x - eqn(3)

substitute eqn(3) in eqn(2)

79750 = 0.1x +0.09y

79750=0.1x + 0.09(900000-3x)

79750=0.1x+ 81000 - 0.27x

collect like terms

79750 -81000 = 0.1x-0.27x

-1250 = -0.17x

to find x divide both sides by -0.17

x = -1250/-0.17 ~= 7353

since 2x = z

2*7353 = 14706

in eqn(3)

y = 900000-3x

y= 900000-3(7353)

y = 900000-22059

y = 877941

x =7353,y= 877941,z=14706

What is the product of the expression, 5x(x2)? (1 point)
25x2
10x
5x3
5x2

Answers

The product of the expression 5x(x²) is 5x³.


1. Write down the given expression: 5x(x²)
2. Apply the distributive property, which states that a(b + c) = ab + ac. In this case, we have a single term inside the parentheses, so the expression becomes: 5x * x²
3. Multiply the coefficients (numbers) together: 5 * 1 = 5
4. Multiply the variables together, which means adding the exponents since they have the same base (x): x¹* x² = x⁽¹⁺²⁾ = x³
5. Combine the result from steps 3 and 4: 5x³

The product of the expression 5x(x²) can be found by multiplying the coefficients (numbers) and adding the exponents of the variables (letters). In this case, we have 5 times x times x squared.

5 times x equals 5x, and x squared means x times x, so we can rewrite the expression as:

5x(x²) = 5x(x*x) = 5x³

So, the product of the expression 5x(x²) is 5x³.

Know more about expression here:

https://brainly.com/question/14083225

#SPJ11

Suppose that in 1682, a man bought a diamond for $32. Suppose that the man had instead put the $32 in the bank at 3% interest compounded continuously. What would that $32 have been worth in 2003? In 2003, the $32 would have been worth $ (Do not round until the final answer. Then round to the nearest dollar as needed.)

Answers

If a man bought a diamond for $32 in 1682 and the man had instead put the $32 in the bank at 3% interest compounded continuously, then the value of the diamond in 2003 would be $554,311.

The given problem is related to exponential growth. In this problem, the continuous compounding formula will be used to find the value of $32 in 2003.

The formula for continuous compounding is given by:

A = Pert   Where,

P is the principal amount,

r is the annual interest rate,

e is the Euler's number which is approximately 2.71828, and

t is the time in years.

Using the formula, we get:

A = 32e^(0.03 x 321)

A = 32e^9.63

A = 32 x 17322.23

A = $ 554311.36

Thus, $32 invested at 3% compounded continuously from 1682 to 2003 would be worth $554,311.

To know more about interest, click on the link below:

https://brainly.com/question/30393144#

#SPJ11

Other Questions
PLEASE SHOW ALL YOUR WORK AS NEATLY AS POSSIBLE: 1) Given f(x) = 3sqrt(x + 2)^2 a) Find the derivative, f'(x). b) Solve f'(x) = 0 Often the unconditioned response and conditioned response look the same(A) True(B) False Consider the tube and inlet conditions of Problem 1. 30 Heat transfer at a rate of 3. 89 MW is delivered to the tube. For an exit pressure of p 8 bar, determine (a) the temperature of the water at the outlet as well as the change in (b) combined thermal and flow work, (c) mechanical energy, and (d) total energy of the water from the inlet to the outlet of the tube. Hint: As a first estimate, neglect the change in mechanical energy in solving part (a). Relevant properties may be obtained from a thermodynamics text Select the correct answer from each drop-down menu.Consider the expression below. 1. what did benjamin constant mean by unlimited sovereignty? why did he fear it? the diagram below shows all the possible totals from adding the results of rolling a two fair dice.a) whats the probability of rolling a total of 5? give ur answer as a fraction in its simplest form.b) if you rolled a pair of fair dice 180 times, how many times would you expect to roll a total of five? I need help ASAP PLEASE! Z-score questionThe weights of logs in a wood pile are normally distributed with a mean of 17 pounds and a standard deviation of 3. 4 pounds v=? image attached please help Question Which of the following is not a cardinal sign of Parkinson's disease? A person climbs to the top of the tallest mountain in South America. Since they are farther away from the center of the Earth, how does their weight compare to what it was at the base of the mountain? From The Enchanted AprilMrs. Wilkins longed to get up and open the shutters, but where she was was really so very delicious. She gave a sigh of contentmentand went on lying there looking round her, taking in everything in her room, her own little room, her very own to arrange just as she pleasedfor this one blessed month, her room bought with her own savings, thefruit of her careful denials, whose door she could bolt if she wanted to. and nobody had the right to come in. It was such a strange little room, so different from any she had known, and so sweet. [. ]Well, this was delicious, to lie there thinking how happy she was, but outside those shutters it was more delicious still. She jumped up. pulled on her slippers, for there was nothing on the stone floor but one small rug, ran to the window and threw open the shutters. "Oh!" cried Mrs. Wilkins. All the radiance of April in Italy lay gathered together at her feet. The sun poured in on her. The sea lay asleep in it, hardly stirring. Across the bay the lovely mountains, exquisitely different in colour, were asleep too in the light and underneath her window, at the bottom ofthe flower-starred grass slope from which the wall of the castle rose up, was a great cypress, cutting through the delicate blues and violets androse-colours of the mountains and the sea like a great black sword. She stared. Such beauty, and she there to see it. Such beauty, and she alive to feel it. Her face was bathed in light. Lovely scents cameup to the window and caressed her. A tiny breeze gently lifted her hair. Far out in the bay a cluster of almost motionless fishing boats hoveredWhat do Mrs. Wilkins's surroundings reveal about her character?O 1. Mrs. Wilkins's new surroundings reveal her distaste for the countryside. 02. Mrs. Wilkins's old surroundings reflect her natural capacity for enjoyment. O 3. Mrs. Wilkins's new surroundings reflect her release from an emotional burden. 04. Mrs. Wilkins's old surroundings reveal her delight in unselfishly serving others. A trip to white mountains of new hampshire from boston will take you 2 3/4 hours. assume you have traveled 1/11 of the way. how much longer will the trip take? Why does Aramis react the way he does to d'Artagnan? On the curve y = x3, point p has the coordinates (2, 8). what is the slope of the curve at point p? Drag each number to the correct location. classify each number according to its value. 4. 2 10-6 2. 1 10-3 3. 1 10-2 3. 2 10-5 3. 5 10-4 5. 8 10-3 5. 2 10-4. look at the blocks of time below for WWII. You need to write TWO paragraphs for each. One paragraph should be about what was happening at the time in the European Theatre and the other should be about what was happening at the time in the Pacific Theatre. Be sure to be very detailed! What country was doing what is necessary! Who was fighting whom, include key events, etc. Blocks of time to document, a paragraph for European Theatre and Pacific Theatre.1939 to December 1941December 7, 1941 to June 1944June 6, 1944 to September 1945 Which of the following is not a current world conflict caused by racial tensions?A.Hutus and TutsisB.suffrageC.Shia and SunniD.apartheid what is subsistence crisis. mention any two factors responsible for this in france? The Great Eastern hotel, a privately owned, independent, five-star deluxe hotel located in Hong Kongs commercial district, isfaced with a turbulent external environment owing to the current financial crisis. Since its inception six years ago, the hotelhas grown in prominence during the bustling economy from 2003 to 2007, and it has been one of the top performers in theupscale and luxury market segments over the four years preceding the economic crises. The hotels main target marketsegment is the business traveler (75 percent of room bookings) who has no problem with paying USD 350 per night for aroom. During the past six months, however, the hotel has been a victim of the severe economic upheaval, which has led to asignificant reduction in room bookings from the business travel segment. This has reduced profits significantly to the extentthat the hotel is no longer able to cover fixed costs. The owner, Jerry Kong, has called an executive committee meeting todiscuss the future direction the company should take in the immediate term and in the long term to sustain its competitiveadvantage. 1. What issues should Jerry and the executive committee address? Why? (Hint: Make assumptions where necessary,including mission and vision statements, as well as goals, strategies, and objectives. )2. Given the preceding information, what are Jerrys options? How should they be evaluated? Make assumptions wherenecessary. 3. What should the hotel do in the short term and in the long term? Make assumptions where necessary to arrive at yourdecisions. 4. Why is it difficult to answer the preceding questions? Do we have clear answers for issues and challenges in real life?5. Do managers and executives in hospitality and tourism organizations always have sufficient and reliable information tomake decisions I need a Persuasive speech it doesnt matter what its about